Convergence of alternating series

Click For Summary
The discussion revolves around proving the convergence of an alternating series and the relationship between its sum and the n-th partial sum. Participants express confusion about how to approach the problem, particularly regarding the conditions needed for the inequality |s - sn| ≤ an+1 to hold true. A counterexample is presented to challenge the validity of the statement, highlighting the importance of assuming that the sequence an is nonnegative and monotonically decreasing to zero. Suggestions are made to analyze the difference between two partial sums to demonstrate that the series is Cauchy. The conversation emphasizes the need for clarity in the assumptions and steps required to prove the convergence of the series.
vikkisut88
Messages
34
Reaction score
0

Homework Statement


Let s be the sum of the alternating series \sum(from n=1 to \infty)(-1)n+1an with n-th partial sum sn. Show that |s - sn| \leqan+1



Homework Equations


I know about Cauchy sequences, the Ratio test, the Root test


The Attempt at a Solution


I really have no idea where I'm meant to start with this question. I didn't know whether I thne have to take more partial sums i.e. partial sums of the n-th partial sum and work with it from there, or something else. I would be really grateful if someone could help me out.
 
Physics news on Phys.org
I don't think your statement is true as you gave it.
For example, let
\begin{cases}<br /> a_0 &amp; {} = 1 \\<br /> a_1 &amp; {} = 0 \\<br /> a_n &amp; {} = 2^{-n} \qquad (n &gt; 1)<br /> \end{cases}

Then s = 3/2, |s - s0| = |5/6 - 1| = |-1/6| = 1/6 which is not smaller than a1 = 0.

[edit]I had s = 5/6, I think it should be 3/2. Doesn't affect my argument though.[/edit]
 
Last edited:
sorry i don't really understand that - how did you work out that s was 5/6? And did you just choose random values for a0, a1 and an? I have rechecked my homework question and that is exactly what it said!
 
CompuChip just gave you a counterexample. To prove your conclusion, you also need to assume that a_n is a nonnegative sequence decreasing to zero monotonically. That might be what you are missing.
 
okay, but i still don't understand how I'm meant to show the result, sorry. This question has got me completely flummoxed.
 
Start by looking at the difference of two partial sums |s_m-s_n| for m<n. Try to show that's less than or equal to a_m+1. Write down the terms making up the difference and regroup them. This would show the sequence is Cauchy, right?
 
Question: A clock's minute hand has length 4 and its hour hand has length 3. What is the distance between the tips at the moment when it is increasing most rapidly?(Putnam Exam Question) Answer: Making assumption that both the hands moves at constant angular velocities, the answer is ## \sqrt{7} .## But don't you think this assumption is somewhat doubtful and wrong?

Similar threads

  • · Replies 1 ·
Replies
1
Views
2K
  • · Replies 7 ·
Replies
7
Views
2K
Replies
7
Views
2K
  • · Replies 3 ·
Replies
3
Views
1K
Replies
14
Views
2K
  • · Replies 2 ·
Replies
2
Views
2K
  • · Replies 4 ·
Replies
4
Views
2K
  • · Replies 5 ·
Replies
5
Views
2K
  • · Replies 3 ·
Replies
3
Views
2K
  • · Replies 2 ·
Replies
2
Views
2K